L&D 1

Réussis tes devoirs et examens dès maintenant avec Quizwiz!

A new mother asks the nurse when the soft spot on her sons head will go away. The nurses answer is based on the knowledge that the anterior fontanel closes after birth by: a. 2 months b. 8 months c. 12 months d. 18 months

18 months The larger of the two fontanels, the anterior fontanel closes by 18 months after birth. The posterior fontanel closes at 6 to 8 weeks.

When assessing the relative advantages of internal electronic fetal monitoring (EFM), nurses should be cognizant of which of the following clients is not an appropriate choice for this type of fetal surveillance: a. A client who still has intact membranes b. A woman whose fetus is well engaged in the pelvis c. A pregnant woman who has a comorbidity of obesity d. A client whose cervix is dilated to 4 to 5 cm

A client who still has intact membranes For internal monitoring, the membranes must have ruptured and the cervix must be sufficiently dilated. The presenting part must be low enough to allow placement of the spiral electrode necessary for internal monitoring. The accuracy of internal monitoring is not affected by maternal size. It may be more difficult to evaluate fetal well-being using external EFM on an obese client. This client is indeed a candidate for internal monitoring. The cervix must be at least 2 to 3 cm dilated.

Which collection of risk factors will most likely result in damaging lacerations (including episiotomies)? a. A dark-skinned woman who has had more than one pregnancy, who is going through prolonged second-stage labor, and who is attended by a midwife b. A reddish-haired mother of two who is going through a breech birth c. A dark-skinned first-time mother who is going through a long labor d. A first-time mother with reddish hair whose rapid labor was overseen by an obstetrician

A first-time mother with reddish hair whose rapid labor was overseen by an obstetrician Reddish-haired women have tissue that is less distensible than darker-skinned women and so may have less efficient healing. First-time mothers are also more at risk, especially with breech births, long second-stage labors, or rapid labors in which there is insufficient time for the perineum to stretch. The rate of episiotomies is higher when obstetricians rather than midwives attend births. This woman is at low risk for either damaging lacerations or an episiotomy. The client is multiparous, has dark skin, and is being attended by a midwife (who is less likely to perform an episiotomy). Reddish-haired women have tissue that is less distensible than that of darker-skinned women. This puts the client at increased risk for lacerations; however, she has had two previous deliveries, which result in a lower likelihood of an episiotomy. The fact that this woman is experiencing a prolonged labor might increase her risk for lacerations. Fortunately she is dark skinned, which indicates that her tissue is more distensible than that of fair-skinned women and therefore less susceptible to injury.

During labor a fetus with an average fetal heart rate (FHR) of 135 beats/min over a 10-minute period is considered to have: a. Bradycardia b. A normal baseline heart rate c. Tachycardia d. Hypoxia

A normal baseline heart rate The baseline heart rate is measured over 10 minutes; a normal range is 110 to 160 beats/min. Bradycardia is a FHR less than 110 beats/min for 10 minutes or longer. Tachycardia is a FHR more than 160 beats/min for 10 minutes or longer. Hypoxia is an inadequate supply of oxygen; no indication of this condition exists with a baseline heart rate in the normal range.

A nurse caring for a woman in labor should understand that absent or minimal variability is classified as either abnormal or indeterminate. Which condition related to decreased variability is considered benign? a. A periodic fetal sleep state b. Extreme prematurity c. Fetal hypoxemia d. Preexisting neurologic injury

A periodic fetal sleep state When the fetus is temporarily in a sleep state there is minimal variability present. Periodic fetal sleep states usually last no longer than 30 minutes. A woman who presents in labor with extreme prematurity may display a fetal heart rate (FHR) pattern of minimal or absent variability. Abnormal variability may also be related to fetal hypoxemia and metabolic acidemia. Congenital anomalies or preexisting neurologic injury may also present as absent or minimal variability. Other possible causes might be central nervous system (CNS) depressant medications, narcotics, or general anesthesia.

A nurse caring for a laboring woman is cognizant that early decelerations are caused by: a. Altered fetal cerebral blood flow b. Umbilical cord compression c. Uteroplacental insufficiency d. Spontaneous rupture of membranes

ANS: A Early decelerations are the fetuss response to fetal head compression. These are considered benign and interventions are not necessary. Variable decelerations are associated with umbilical cord compression. Late decelerations are associated with uteroplacental insufficiency. Spontaneous rupture of membranes has no bearing on the fetal heart rate (FHR) unless the umbilical cord prolapses, which would result in variable or prolonged bradycardia.

Fetal tachycardia is most common during: a. Maternal fever b. Umbilical cord prolapse c. Regional anesthesia d. Magnesium sulfate administration

ANS: A Fetal tachycardia can be considered an early sign of fetal hypoxemia and may also result from maternal or fetal infection. Umbilical cord prolapse will most likely result in fetal bradycardia, not tachycardia. Regional anesthesia will most likely result in fetal bradycardia, not tachycardia. Magnesium sulfate administration will most likely result in fetal bradycardia, not tachycardia.

Women who have participated in childbirth education classes often bring a birth plan with them to the hospital. This document includes the couples preference related to (choose all that apply): a. Presence of companions b. Clothing to be worn c. Care and handling of the newborn d. Medical interventions e. Environmental modifications

ANS: A, B, C, D, E The presence of companions, clothing to be worn, care and handling of the newborn, medical interventions, and environmental modifications all might be included in the couples birth plan. Other items include the presence of nonessential medical personnel (students), labor activities such as the tub or ambulation, preferred comfort and relaxation methods, and any cultural or religious requirements. Couples can visit www.childbirth.org for an interactive birth plan to assist them in preparing for their birth.

Emergency conditions during labor that require immediate nursing intervention can arise with startling speed. Examples of such emergencies include (choose all that apply): a. Nonreassuring or abnormal fetal heart rate (FHR) pattern b. Inadequate uterine relaxation c. Vaginal bleeding d. Infection e. Prolapse of the cord

ANS: A, B, C, D, E A nonreassuring or abnormal FHR pattern, inadequate uterine relaxation, vaginal bleeding, infection, and cord prolapse all constitute an emergency during labor that requires immediate nursing intervention.

Complications and risks associated with cesarean births include (choose all that apply): a. Pulmonary edema b. Wound dehiscence c. Hemorrhage d. Urinary tract infections e. Fetal injuries

ANS: A, B, C, D, E Pulmonary edema, wound dehiscence, hemorrhage, urinary tract infections, and fetal injuries are possible complications and risks associated with cesarean births.

In assessing the immediate condition of the newborn after birth, a sample of cord blood may be a useful adjunct to the Apgar score. Cord blood is then tested for pH, carbon dioxide, oxygen, and base deficit or excess. Clinical situations that warrant this additional testing include (choose all that apply): a. Low 5-minute Apgar score b. Intrauterine growth restriction (IUGR) c. Maternal thyroid disease d. Intrapartum fever e. Multiple gestation f. Abnormal fetal heart rate tracing

ANS: A, B, C, D, E, F The American College of Obstetricians and Gynecologists (ACOG) suggests obtaining cord blood values in all of these clinical situations. Samples can be drawn from both the umbilical artery and the umbilical vein. Results may indicate that fetal compromise has occurred.

The role of the nurse with regard to informed consent is to: a. Inform the client about the procedure and have her sign the consent form b. Act as a client advocate and help clarify the procedure and the options c. Call the physician to see the client d. Witness the signing of the consent form

Act as a client advocate and help clarify the procedure and the options Nurses play a part in the informed consent process by clarifying and describing procedures or by acting as the womans advocate and asking the primary health care provider for further explanations. The physician is responsible for informing the woman of her options, explaining the procedure, and advising the client about potential risk factors. The physician must be present to explain the procedure to the client. However, the nurses responsibilities go further than simply asking the physician to see the client. The nurse may witness the signing of the consent form. However, depending on the states guidelines, the womans husband or another hospital health care employee may sign as witness.

Nurses can help their clients by keeping them informed about the distinctive stages of labor. What description of the phases of the first stage of labor is accurate? a. Latent: mild, regular contractions; no dilation; bloody show b. Active: moderate, regular contractions; 4 to 7 cm dilation c. Lull: no contractions; dilation stable d. Transition: very strong but irregular contractions; 8 to 10 cm dilation

Active: moderate, regular contractions; 4 to 7 cm dilation The active phase is characterized by moderate, regular contractions, 4 to 7 cm dilation, and a duration of 3 to 6 hours. The latent phase is characterized by mild to moderate, irregular contractions, dilation up to 3 cm, brownish to pale pink mucus, and a duration of 6 to 8 hours. No official lull phase exists in the first stage. The transition phase is characterized by strong to very strong, regular contractions, 8 to 10 cm dilation, and a duration of 20 to 40 minutes.

A woman in labor is breathing into a mouthpiece just before the start of her regular contractions. As she inhales, a valve opens and gas is released. She continues to inhale the gas slowly and deeply until the contraction starts to subside. When the inhalation stops, the valve closes. This procedure is: a. Not used much anymore b.Likely to be used in the second stage of labor but not in the first stage c. An application of nitrous oxide d. A prelude to cesarean birth

An application of nitrous oxide This is an application of nitrous oxide, which can be used in combination with other nonpharmacologic and pharmacologic measures for pain relief. This is an application of nitrous oxide mixed with oxygen in a low concentration. It is still commonly used in Canada and the United Kingdom. Nitrous oxide inhaled in a low concentration will reduce but not eliminate pain during the first and second stages of labor. Nitrous oxide inhalation is not generally used prior to caesarean birth. Nitrous oxide does not appear to depress uterine contractions or cause adverse reactions in the newborn.

Concerning the third stage of labor, nurses should be aware that: a. The placenta eventually detaches itself from a flaccid uterus b. An expectant or active approach to managing this stage of labor reduces the risk of complications c. It is important that the dark, roughened maternal surface of the placenta appears before the shiny fetal surface d. The major risk for women during the third stage is a rapid heart rate

An expectant or active approach to managing this stage of labor reduces the risk of complications Active management facilitates placental separation and expulsion, reducing the risk of complications. The placenta cannot detach itself from a flaccid (relaxed) uterus. Which surface of the placenta comes out first is not clinically important. The major risk for women during the third stage of labor is postpartum hemorrhage.

Nurses should be aware that accelerations in the fetal heart rate (FHR): a. Are indications of fetal well-being when they are periodic b. Are greater and longer in preterm gestations c. Are usually seen with breech presentations when they are episodic d. Are a visually apparent abrupt peak

Are a visually apparent abrupt peak Acceleration of the FHR is defined as a visually apparent abrupt (only to peak 30 seconds) increase in FHR above the baseline rate. Periodic accelerations occur with uterine contractions (UCs) and usually are seen with breech presentations. Episodic accelerations occur during fetal movement and are indications of fetal well-being. Preterm accelerations peak at 10 beats/min above the baseline and last for at least 10 seconds. Periodic accelerations occur with UCs and usually are seen with breech presentations. Episodic accelerations occur during fetal movement and are indications of fetal well-being.

The nurse caring for a woman in labor understands that prolonged decelerations: a. Are a continuing pattern of benign decelerations that do not require intervention b. Constitute a baseline change when they last longer than 5 minutes c. Are caused by a disruption to the fetal O2 supply d. Require the usual fetal monitoring by the nurse

Are caused by a disruption to the fetal O2 supply Prolonged decelerations are caused by a disruption in the fetal oxygen supply. They usually begin as a reflex response to hypoxia. If the disruption continues, the fetal cardiac tissue itself will become hypoxic, resulting in direct myocardial depression of the FHR. These can be caused by prolonged cord compression, uteroplacental insufficiency, or perhaps sustained head compression. Prolonged decelerations lasting more than 10 minutes are considered a baseline change that may require intervention. A prolonged deceleration is a visually apparent decrease (may be either gradual or abrupt) in fetal heart rate (FHR) of at least 15 beats/min below the baseline and lasting more than 2 minutes but less than 10 minutes. Nurses should notify the physician or nurse-midwife immediately and initiate appropriate treatment of abnormal patterns when they see a prolonged deceleration.

The nurse understands that approaches to relaxation in labor include (choose all that apply): a. Aromatherapy b. Massage c. Hypnosis d. Cesarean birth e. Biofeedback

Aromatherapy Massage Hypnosis Biofeedback Approaches to relaxation can include neuromuscular relaxation, aromatherapy, music, massage, imagery, hypnosis, or touch relaxation. Cesarean birth is a method of delivery, not a method of relaxation.

When a nulliparous woman telephones the hospital to report that she is in labor, the nurse initially should: a. Tell the woman to stay home until her membranes rupture b. Emphasize that food and fluid intake should stop c. Arrange for the woman to come to the hospital for labor evaluation d. Ask the woman to describe why she believes she is in labor

Ask the woman to describe why she believes she is in labor Assessment begins at the first contact with the woman, whether by telephone or in person. By asking the woman to describe her signs and symptoms, the nurse can begin her assessment and gather data. The initial nursing activity should be to gather data about the womans status. The amniotic membranes may or may not spontaneously rupture during labor. The client may be instructed to stay home until the uterine contractions become strong and regular. The initial nursing activity should be to gather data about the womans status. After this assessment has been made, the nurse may want to discuss the appropriate oral intake for early labor, such as light foods or clear liquids, depending on the preference of the client or her primary health care provider. Before instructing the woman to come to the hospital, the nurse should initiate her assessment during the telephone interview.

A nurse providing care for a woman with preterm labor on terbutaline includes which intervention to identify side effects of the drug? a. Assess deep tendon reflexes (DTRs). b. Assess for dyspnea and crackles. c. Assess for bradycardia. d. Assess for hypoglycemia.

Assess for dyspnea and crackles. Terbutaline is a beta2-adrenergic agonist that affects the mothers cardiopulmonary and metabolic systems. Signs of cardiopulmonary decompensation include adventitious breath sounds and dyspnea. Assessing DTRs does not address the possible respiratory side effects of using terbutaline. It is important for the nurse to perform this assessment if the woman is on magnesium sulfate. Terbutaline is a beta2-adrenergic agonist that affects the mothers cardiopulmonary and metabolic systems. Beta2-adrenergic agonist drugs cause tachycardia, not bradycardia. Terbutaline is a beta2-adrenergic agonist that affects the mothers cardiopulmonary and metabolic systems. The metabolic effect leads to hyperglycemia, not hypoglycemia.

A client is to have an amniotomy to induce labor. The nurse recognizes that the priority intervention after the amniotomy is to: a. Apply clean linens under the woman b. Take the clients vital signs c. Perform a vaginal examination d. Assess the fetal heart rate (FHR)

Assess the fetal heart rate (FHR) The FHR is assessed before and immediately after the amniotomy to detect any changes that might indicate cord compression or prolapse. Providing comfort measures for the client is important, but it is not the priority immediately after an amniotomy. The womans temperature should be checked every 2 hours after rupture of membranes, but this is not the priority at this time. The woman would have had a vaginal examination during the procedure. Unless cord prolapse is suspected, a vaginal examination is not warranted. Additionally, FHR assessment provides clinical cues to a prolapsed cord.

A multiparous woman has been in labor for 8 hours. Her membranes have just ruptured. The nurses initial response is to: a. Prepare the woman for imminent birth b. Notify the womans primary health care provider c. Document the characteristics of the fluid d. Assess the fetal heart rate and pattern

Assess the fetal heart rate and pattern The umbilical cord may prolapse when the membranes rupture. The fetal heart rate and pattern should be monitored closely for several minutes immediately after ROM to ascertain fetal well-being, and the findings should be documented. Rupture of membranes (ROM) may increase the intensity and frequency of the uterine contractions, but it does not indicate that birth is imminent. The nurse may notify the primary health care provider after ROM occurs and the fetal well-being and response to ROM have been assessed. The nurses priority is to assess fetal well-being. The nurse should document the characteristics of the amniotic fluid, but the initial response is to assess fetal well-being and the response to ROM.

A woman who is pregnant for the first time is dilated 3 cm and having contractions every 5 minutes. She is groaning and perspiring excessively; she states that she did not attend childbirth classes. The most important nursing action is to: a. Notify the womans health care provider b. Administer the prescribed narcotic analgesic c. Assure her that her labor will be over soon d. Assist her with simple breathing and relaxation instructions

Assist her with simple breathing and relaxation instructions By reducing tension and stress, focusing and relaxation techniques allow the woman in labor to rest and conserve energy for the task of giving birth. For those who have had no preparation, instruction in simple breathing and relaxation can be given in early labor and often is successful. The nurse can perform many functions in labor and birth independently, such as teaching and support. Pain medication may be an option for this client. However, the initial response of the nurse should include educating the client about her options. The length of labor varies among individuals, but the first stage of labor is the longest. At 3 cm of dilation with contractions every 5 minutes, this woman has a significant amount of labor yet to experience.

When assessing a woman in labor, the nurse is aware that the relationship of the fetal body parts to one another is called fetal: a. Lie b. Presentation c. Attitude d. Position

Attitude Attitude is the relation of the fetal body parts to each other. Lie is the relation of the long axis (spine) of the fetus to the long axis (spine) of the mother. Presentation refers to the part of the fetus that enters the pelvic inlet first and leads through the birth canal during labor at term. Position is the relation of the presenting part to the four quadrants of the mothers pelvis.

Five essential components of any fetal heart rate (FHR) tracing must be evaluated regularly. These include (choose all that apply): a. Baseline rate b. Baseline variability c. Accelerations d. Decelerations e. Changes or trends over time f. Frequency of contractions

Baseline rate Baseline variability Accelerations Decelerations Changes or trends over time The five essential components of the FHR tracing that must be evaluated regularly are baseline rate, baseline variability, accelerations, decelerations, and changes or trends over time. Whenever one of these five essential components is assessed as abnormal, corrective measures must immediately be taken. Contraction frequency is not an essential component of the FHR tracing.

Obese women are at risk for several complications during pregnancy and birth. These include all except: a. Thromboembolism b. Cesarean birth c. Wound infection d. Breech presentation

Breech presentation A breech presentation is not a complication of pregnancy or birth for the obese client. Venous thromboembolism is a known risk for obese women. The use of TED hose and sequential compression devices may help to decrease the chance for clot formation. Women should also be encouraged to ambulate as soon as possible. In addition to having an increased risk for cesarean birth in general, obese women are also more likely to require emergency cesarean birth. Many obese women have a pannus (large roll of abdominal fat) that overlies a lower transverse incision made just above the pubic area. The pannus causes the area to remain moist, which encourages the development of infection.

Which characteristic correctly matches the type of deceleration with its likely cause? a. Early decelerationumbilical cord compression b. Late decelerationuteroplacental insufficiency c. Variable decelerationhead compression d. Prolonged decelerationcause unknown

Late decelerationuteroplacental insufficiency Late deceleration is caused by uteroplacental insufficiency. Early deceleration is caused by head compression. Variable deceleration is caused by umbilical cord compression. Prolonged deceleration has a variety of either benign or critical causes.

Which statement related to cephalopelvic disproportion (CPD) is the least accurate? a. CPD can be related to either fetal size or fetal position. b. The fetus cannot be born vaginally. c. CPD can be accurately predicted. d. The cause may be of maternal or fetal origin.

CPD can be accurately predicted. Unfortunately, there is no way to accurately predict CPD. Although CPD is often related to excessive fetal size or macrosomia, the problem in many cases is malposition of the fetal presenting part rather than true CPD. When CPD is present, the fetus cannot fit through the maternal pelvis to be born vaginally. CPD may be related to either fetal origins such as macrosomia or malposition or maternal origins such as a too small or malformed pelvis.

Which presentation is described accurately in terms of both presenting part and frequency of occurrence? a. Cephalic: occiput; at least 96% b. Breech: sacrum; 10% to 15% c. Shoulder: scapula; 10% to 15% d. Cephalic: cranial; 80% to 85%

Cephalic: occiput; at least 96% In cephalic presentations (head first) the presenting part is the occiput; this occurs in 96% of births. In a breech birth the sacrum emerges first; this occurs in about 3% of births. In shoulder presentations the scapula emerges first; this occurs in only 1% of births. In a cephalic presentation the part of the head or cranium that emerges first is the occiput; cephalic presentations occur in 96% of births.

While evaluating an external monitor tracing of a woman in active labor, the nurse notes that the fetal heart rate (FHR) for five sequential contractions begins to decelerate late in the contraction, with the nadir of the decelerations occurring after the peak of the contraction. The nurses first priority is to: a. Change the womans position b. Notify the health care provider c. Assist with amnioinfusion d. Insert a scalp electrode

Change the womans position Late decelerations may be caused by maternal supine hypotension syndrome. They usually are corrected when the woman turns onto her side to displace the weight of the gravid uterus from the vena cava. If the fetus does not respond to primary nursing interventions for late decelerations, the nurse should continue with subsequent intrauterine resuscitation measures, including notifying the health care provider. An amnioinfusion may be used to relieve pressure on an umbilical cord that has not prolapsed. The fetal heart rate pattern associated with this situation most likely will reveal variable decelerations. A fetal scalp electrode provides accurate data for evaluating the well-being of the fetus; however, this is not a nursing intervention that will alleviate late decelerations, nor is it the nurses first priority.

A normal uterine activity (UA) pattern in labor is characterized by: a. Contractions every 2 to 5 minutes b. Contractions lasting about 2 minutes c. Contractions about 1 minute apart d. A contraction intensity of about 500 mm Hg with relaxation at 50 mm Hg

Contractions every 2 to 5 minutes Contraction frequency overall generally ranges from two to five per 10 minutes of labor, with lower frequencies during the first stage and higher frequency seen during the second stage. Contraction duration remains fairly stable throughout the first and second stages, ranging from 45 to 80 seconds, not generally exceeding 90 seconds. Contractions 1 minute apart are too often, and this would be considered an abnormal labor pattern. Intensity of uterine contractions generally ranges from 25 to 50 mm Hg in the first stage of labor and may rise to more than 80 mm Hg in the second stage.

In evaluating the effectiveness of an oxytocin induction, the nurse expects: a. Contractions lasting 40 to 90 seconds, 2 to 3 minutes apart b. The intensity of contractions to be at least 110 to 130 mm Hg c. Labor to progress at least 2 cm/hr dilation d. At least 30 milliunits/min of oxytocin are needed to achieve cervical dilation

Contractions lasting 40 to 90 seconds, 2 to 3 minutes apart The goal of induction of labor is to produce contractions that occur every 2 to 3 minutes and last 80 to 90 seconds. The intensity of the contractions should be 40 to 90 mm Hg by IUPC. Cervical dilation of 1 cm/hr in the active phase of labor is the goal in an oxytocin induction. The dose is increased by 1 to 2 milliunits/min at intervals of 30 to 60 minutes until the desired contraction pattern is achieved. Doses are increased up to a maximum of 20 to 40 milliunits/min.

Perinatal nurses are legally responsible for: a. Correctly interpreting fetal heart rate (FHR) patterns, initiating appropriate nursing interventions, and documenting the outcomes b. Greeting the client on arrival, assessing her, and starting an IV line c. Applying the external fetal monitor and notifying the health care provider d. Making sure the woman is comfortable

Correctly interpreting fetal heart rate (FHR) patterns, initiating appropriate nursing interventions, and documenting the outcomes Nurses who care for women during childbirth are legally responsible for correctly interpreting FHR patterns, initiating appropriate nursing interventions based on those patterns, and documenting the outcomes of those interventions. Greeting the client on arrival, assessing her, and starting an IV line are activities that should be performed when any client arrives to the maternity unit. The nurse is not the only one legally responsible for performing these functions. Applying the external fetal monitor and notifying the health care provider is a nursing function that is part of the standard of care for all obstetric clients. This falls within the RN scope of practice. Everyone caring for the pregnant woman should ensure that both she and her support partner are comfortable.

Nurses with an understanding of cultural differences regarding likely reactions to pain may be better able to help clients. Nurses should know that _____ women may be stoic until late in labor, when they may become vocal and request pain relief. a. Chinese b. Arab or Middle Eastern c. Hispanic d. African-American

Hispanic Hispanic women may be stoic early, more vocal and ready for medications later. Chinese women may not show reactions to pain. Medical interventions must be offered more than once. Arab or Middle Eastern women may be vocal in response to labor pain from the start. They may prefer pain medications. African-American women may express pain openly; use of medications for pain is more likely to vary with the individual.

When assessing a woman in the first stage of labor, the nurse recognizes that the most conclusive sign that uterine contractions are effective is: a. Dilation of the cervix b. Descent of the fetus c. Rupture of the amniotic membranes d. Increase in bloody show

Dilation of the cervix The vaginal examination reveals whether the woman is in true labor. Cervical change, especially dilation, in the presence of adequate labor indicates that the woman is in true labor. Descent of the fetus, or engagement, may occur before labor. Rupture of membranes may occur with or without the presence of labor. Bloody show may indicate slow, progressive cervical change (e.g., effacement) in both true and false labor.

The nurse who performs vaginal examinations to assess a womans progress in labor should: a. Perform an examination at least once every hour during the active phase of labor b. Perform the examination with the woman in the supine position c. Wear two clean gloves for each examination d. Discuss the findings with the woman and her partner

Discuss the findings with the woman and her partner The nurse should discuss the findings of the vaginal examination with the woman and her partner and report them to the primary care provider. A vaginal examination should be performed only when indicated by the status of the woman and her fetus. The woman should be positioned so as to avoid supine hypotension. The examiner should wear a sterile glove while performing a vaginal examination for a laboring woman.

The nurse is teaching a client with PPROM about self-care activities. Which activities should the nurse include in her teaching? Choose all that apply. a. Report a temperature higher than 40 C. b. It is safe to use a tampon to absorb the leaking amniotic fluid. c. Do not engage in sexual activity. d. It is safe to take frequent tub baths. e. It is acceptable to douche to cleanse the vagina of foul-smelling fluids.

Do not engage in sexual activity. Sexual activity should be avoided because it may induce preterm labor. A temperature of more than 38 C should be reported. Tub baths should be avoided to prevent the risk of infection. To avoid the risk of infection, nothing should be inserted into the vagina. Furthermore, foul-smelling vaginal fluid should be reported because this may be a sign of infection.

As a perinatal nurse, you realize that a fetal heart rate (FHR) that is tachycardic, bradycardic, has late decelerations, or loss of variability is nonreassuring and is associated with: a. Hypotension b. Cord compression c. Maternal drug use d. Hypoxemia

Hypoxemia Nonreassuring heart rate patterns are associated with fetal hypoxemia. Fetal bradycardia may be associated with maternal hypotension. Fetal variable decelerations are associated with cord compression. Maternal drug use is associated with fetal tachycardia.

The uterine contractions of a woman early in the active phase of labor are assessed by an internal uterine pressure catheter (IUPC). The nurse notes that the intrauterine pressure at the peak of the contraction ranges from 65 to 70 mm Hg, and the resting tone range is 6 to 10 mm Hg. The uterine contractions occur every 3 to 4 minutes and last an average of 55 to 60 seconds. Based on this information, the nurse should: a. Notify the womans primary health care provider immediately b. Prepare to administer an oxytocic to stimulate uterine activity c. Document the findings because they reflect the expected contraction pattern for the active phase of labor d. Prepare the woman for the onset of the second stage of labor

Document the findings because they reflect the expected contraction pattern for the active phase of labor The nurse is responsible for monitoring the uterine contractions to ascertain whether they are powerful and frequent enough to accomplish the work of expelling the fetus and the placenta. Additionally the nurse documents these findings in the clients medical record. This labor pattern indicates that the client is in the active phase of the first stage of labor. Nothing indicates a need to notify the primary health care provider at this time. Oxytocin augmentation is not needed for this labor pattern; this contraction pattern indicates adequate active labor. This labor pattern indicates that the woman is in active labor. Her contractions eventually will become stronger, last longer, and come closer together during the transition phase of the first stage of labor. The transition phase precedes the second stage of labor, or delivery of the fetus.

Which deceleration of the fetal heart rate (FHR) does not require the nurse to change the maternal position? a. Early decelerations b. Late decelerations c. Variable decelerations d. It is always a good idea to change the womans position.

Early decelerations Early decelerations (and accelerations) generally do not need any nursing intervention. Late decelerations suggest that the nurse should change the maternal position (lateral). Variable decelerations also require a maternal position change (side to side). Although changing positions throughout labor is recommended, it is not required in response to early decelerations.

Maternity nurses often have to answer questions about the many, sometimes unusual ways people have tried to make the birthing experience more comfortable. For instance, nurses should be aware that: a. Music supplied by the support person has to be discouraged because it could disturb others or upset the hospital routine b. Women in labor can benefit from sitting in a bathtub, but they must limit immersion to no longer than 15 minutes at a time c. Effleurage is permissible, but counterpressure is almost always counterproductive d. Electrodes attached to either side of the spine to provide high-intensity electrical impulses facilitate the release of endorphins

Electrodes attached to either side of the spine to provide high-intensity electrical impulses facilitate the release of endorphins Transcutaneous electrical nerve stimulation (TENS) may help and is most useful for lower back pain that occurs during the first stage of labor. Music may be very helpful for reducing tension and certainly can be accommodated by the hospital. Women can stay in a bath as long as they want, although repeated baths with breaks might be more effective than a long soak. Counterpressure can help the woman cope with lower back pain.

In assisting with the two factors that have an effect on fetal status, namely pushing and positioning, nurses should: a. Encourage the womans cooperation in avoiding the supine position b. Advise the woman to avoid the semi-Fowler position c. Encourage the woman to hold her breath and tighten her abdominal muscles to produce a vaginal response d. Instruct the woman to open her mouth and close her glottis, letting air escape after the push

Encourage the womans cooperation in avoiding the supine position The woman should maintain a side-lying position. The semi-Fowler position is the recommended side-lying position with a lateral tilt to the uterus. Encouraging the woman to hold her breath and tighten her abdominal muscles is the Valsalva maneuver, which should be avoided. Both the mouth and glottis should be open, letting air escape during the push.

Which action is correct when palpation is used to assess the characteristics and pattern of uterine contractions? a. Place the hand on the abdomen below the umbilicus and palpate uterine tone with the fingertips. b. Determine the frequency by timing from the end of one contraction to the end of the next contraction. c. Evaluate the intensity by pressing the fingertips into the uterine fundus. d. Assess uterine contractions every 30 minutes throughout the first stage of labor.

Evaluate the intensity by pressing the fingertips into the uterine fundus. The nurse or primary health care provider may assess uterine activity by palpating the fundal section of the uterus using the fingertips. Many women may experience labor pain in the lower segment of the uterus, which may be unrelated to the firmness of the contraction detectable in the uterine fundus. The frequency of uterine contractions is determined by palpating from the beginning of one contraction to the beginning of the next contraction. Assessment of uterine activity is performed in intervals based on the stage of labor. As labor progresses, this assessment is performed more frequently.

A nulliparous woman who has just begun the second stage of her labor most likely: a. Experiences a strong urge to bear down b. Shows perineal bulging c. Feels tired yet relieved that the worst is over d. Shows an increase in bright red bloody show

Feels tired yet relieved that the worst is over Common maternal behaviors during the latent phase of the second stage of labor include feeling a sense of accomplishment and optimism because the worst is over. The woman may be very quiet during this phase. During the latent phase of the second stage of labor, the urge to bear down often is absent or only slight during the acme of contractions. Perineal bulging occurs during the transition phase of the second stage of labor, not at the beginning of the second stage. An increase in bright red bloody show occurs during the descent phase of the second stage of labor.

A number of methods to assist in the assessment of fetal well-being have been developed for use in conjunction with electronic fetal monitoring (EFM). These various technologies assist in supporting interventions for a nonreassuring fetal heart rate (FHR) pattern when necessary. The labor and delivery nurse should be aware that one of these modalities, fetal oxygen saturation monitoring, includes the use of: a. A fetal acoustic stimulator b. Fetal blood sampling c. Fetal pulse oximetry d. Umbilical cord acid-base determination

Fetal pulse oximetry Continuous monitoring of the fetal O2 saturation by fetal pulse oximetry is a method that was approved for clinical use in 2000 by the U.S. Food and Drug Administration (FDA). This process is similar to obtaining a pulse oximetry in a child or adult. A specially designed sensor is inserted into the uterus and lies against the fetuss temple or cheek. A normal result is 30% to 70%, with 30% being the cutoff for further intervention. Stimulation of the fetus is done in an effort to elicit a FHR response. The two acceptable methods of stimulation are fetal scalp stimulation or vibroacoustic stimulation. Vibroacoustic stimulation is performed by using an artificial larynx or fetal acoustic stimulation device over the fetal head for 1 or 2 seconds. Sampling of the fetal scalp blood was designed to assess fetal pH, O2, and CO2. The sample is obtained from the fetal scalp through a dilated cervix. This test is usually done in tertiary care centers, where results can be immediately available. It has fallen out of favor because test results vary widely.

Certain changes stimulate chemoreceptors in the aorta and carotid bodies to prepare the fetus for initiating respirations immediately after birth. These changes occur naturally during labor and include all except: a. Fetal lung fluid is cleared from the air passages during labor and vaginal birth b. Fetal oxygen pressure decreases (PO2) c. Fetal arterial carbon dioxide increases (PCO2) d. Fetal respiratory movements increase during labor

Fetal respiratory movements increase during labor Fetal respiratory movements actually decrease during labor. Fetal lung fluid is cleared from the air passages during labor and vaginal birth. Fetal oxygen pressure decreases (PO2). Fetal arterial carbon dioxide increases (PCO2).

To adequately care for a laboring woman, the nurse should know which stage of labor varies the most in length? a. First b. Second c. Third d. Fourth

First The first stage of labor is considered to last from the onset of regular uterine contractions to full dilation of the cervix. The first stage is much longer than the second and third stages combined. In a first-time pregnancy the first stage of labor can take up to 20 hours. The second stage of labor lasts from the time the cervix is fully dilated to the birth of the fetus. The average length is 20 minutes for a multiparous woman and 50 minutes for a nulliparous woman. The third stage of labor lasts from the birth of the fetus until the placenta is delivered. This stage may be as short as 3 minutes or as long as 1 hour.

Through vaginal examination, the nurse determines that a woman is 4 cm dilated, and the external fetal monitor shows uterine contractions every 3 to 4 minutes. The nurse reports this as: a. First stage, latent phase b. First stage, active phase c. First stage, transition phase d. Second stage, latent phase

First stage, active phase This maternal progress indicates that the woman is in the active phase of the first stage of labor. During the latent phase of the first stage of labor, the expected maternal progress is 0 to 3 cm dilation with contractions every 5 to 30 minutes. During the transition phase of the first stage of labor, the expected maternal progress is 8 to 10 cm dilation with contractions every 2 to 3 minutes. During the latent phase of the second stage of labor, the woman is completely dilated and experiences a restful period of laboring down.

Which description of the four stages of labor is correct for both definition and duration? a. First stage: onset of regular uterine contractions to full dilation; less than 1 hour to 20 hours b. Second stage: full effacement to 4 to 5 cm; visible presenting part; 1 to 2 hours c. Third stage: active pushing to birth; 20 minutes (multiparous women), 50 minutes (first-timer) d. Fourth stage: delivery of the placenta to recovery; 30 minutes to 1 hour

First stage: onset of regular uterine contractions to full dilation; less than 1 hour to 20 hours Full dilation may occur in less than 1 hour, but in first-time pregnancies it can take up to 20 hours. The second stage extends from full dilation to birth and takes an average of 20 to 50 minutes, although 2 hours is still considered normal. The third stage extends from birth to expulsion of the placenta and usually takes a few minutes. The fourth stage begins after expulsion of the placenta and lasts until homeostasis is reestablished (about 2 hours).

To provide the necessary assessment of parent education, the nurse must know which bone is not a bone in the fetal skull? a. Parietal b. Temporal c. Fontanel d. Occipital

Fontanel The fetal skull has two parietal bones, two temporal bones, an occipital bone, and a frontal bone. The fontanels are membrane-filled spaces. The fetal skull has two parietal bones. The fetal skull has two temporal bones. The fetal skull has an occipital bone and a frontal bone.

A womans position is very important in the progress of labor. While discussing optimal positioning, maternity nurses should be able to tell the client that: a. The supine position commonly used in the United States increases blood flow b. The all fours position, on her hands and knees, is hard on her back c. Frequent changes in position help relieve her fatigue and increase her comfort d. In a sitting or squatting position her abdominal muscles will have to work harder

Frequent changes in position help relieve her fatigue and increase her comfort Frequent position changes relieve fatigue, increase comfort, and improve circulation. Blood flow can be compromised in the supine position; any upright position benefits cardiac output. The all fours position is used to relieve backache in certain situations. In a sitting or squatting position, the abdominal muscles work in greater harmony with uterine contractions.

During anesthetic management of the morbidly obese woman in labor, the nurse must remain alert for complications specific to this type of client. Which is not a concern for the L&D nurse? a. Failed epidural placement b. Accidental dural puncture c. Inadequate pain relief d. Difficult intubation

Inadequate pain relief Use of regional anesthetic can provide adequate pain management for the obese client during labor and delivery. Problems may be encountered when positioning the woman and identifying correct landmarks to ensure proper insertion of the epidural. Due to the difficulty with epidural placement in the obese client, an accidental dural puncture is more likely to occur. During an emergent delivery, anesthesia providers encounter more complications and difficult intubation in the obese parturient.

A woman has requested an epidural for her pain. She is 5 cm dilated and 100% effaced. The baby is in a vertex position and is engaged. The nurse increases the womans IV fluid for a preprocedural bolus. Prior to initiation of the epidural the woman should be informed regarding the disadvantages of an epidural block. They include all except: a. Ability to move freely is limited b. Orthostatic hypotension and dizziness c. Gastric emptying is not delayed d. Higher rate of fever

Gastric emptying is not delayed This is an advantage of an epidural block. Other advantages include the woman remains alert and able to participate, good relaxation is achieved, airway reflexes remain intact, only partial motor paralysis develops and blood loss is not excessive. The womans ability to move freely and to maintain control of her labor is limited, related to the use of numerous medical interventions (IV and electronic fetal monitoring [EFM]). Significant disadvantages of an epidural block include the occurrence of orthostatic hypotension, dizziness, sedation, and leg weakness. Women who receive an epidural have a higher rate of fever (38 C or greater) especially when labor lasts longer than 12 hours. This may result in an unnecessary neonatal workup for sepsis.

Conscious relaxation is associated with which method of childbirth preparation? a. Grantly Dick-Read method b. Lamaze method c. Bradley method d. Psychoprophylactic method

Grantly Dick-Read method With the Grantly Dick-Read method, women are taught to consciously and progressively relax different muscle groups throughout the body until a high degree of skill at relaxation is achieved. The Lamaze method combines controlled muscular relaxation with breathing techniques. The Bradley method advocates natural labor, without any form of anesthesia or analgesia, assisted by a husband-coach and using breathing techniques for labor. The psychoprophylactic method is another name for the Lamaze method.

Nurses should be aware that all reputable childbirth methods attempt to meet all these goals except: a. Increase the womans sense of control b. Prepare a support person to help in labor c. Guarantee a pain-free childbirth d. Learn distraction techniques

Guarantee a pain-free childbirth All methods try to increase a womans sense of control, prepare a support person, and train the woman in physical conditioning, which includes breathing techniques. They cannot, and reputable ones do not, promise a pain-free childbirth. Increasing a womans sense of control is the goal of all childbirth preparation methods. Preparing a support person to help in labor is a vitally important component of any childbirth education program. The coach may learn how to touch a womans body to detect tense and contracted muscles. The woman then learns how to relax in response to the gentle stroking by the coach. Distraction techniques are a form of care that are effective to some degree in relieving labor pain and are taught in many childbirth programs. This includes imagery, feedback relaxation, and attention-focusing behaviors.

In the recovery room, if a woman is asked either to raise her legs (knees extended) off the bed or to flex her knees, place her feet flat on the bed, and raise her buttocks well off the bed, most likely she is being tested to see whether she: a. Has recovered from epidural or spinal anesthesia b. Has hidden bleeding underneath her c. Has regained some flexibility d. Is a candidate to go home after 6 hours

Has recovered from epidural or spinal anesthesia If the numb or prickly sensations are gone from her legs after these movements, she likely has recovered from the epidural or spinal anesthesia. It is always important to assess the client for bleeding beneath her buttocks prior to discharge from the recovery room; however, she should be rolled to her side for this assessment. The nurse is not required to assess the woman for flexibility. This assessment is performed in order to evaluate if the client has recovered from spinal anesthesia, not to determine if she is a candidate for early discharge.

Because the risk for childbirth complications may be revealed, nurses should know that the point of maximal intensity (PMI) of the fetal heart rate (FHR) is: a. Usually directly over the fetal abdomen b. In a vertex position, heard above the mothers umbilicus c. Heard lower and closer to the midline of the mothers abdomen as the fetus descends and rotates internally d. In a breech position, heard below the mothers umbilicus

Heard lower and closer to the midline of the mothers abdomen as the fetus descends and rotates internally Nurses should be prepared for the shift. The PMI of the FHR usually is directly over the fetal back. In a vertex position, it is heard below the mothers umbilicus. In a breech position, it is heard above the mothers umbilicus.

If an opioid antagonist is administered to a laboring woman, she should be told: a. Her pain will decrease b. Her pain will return c. She will feel less anxious d. She will no longer feel the urge to push

Her pain will return Opioid antagonists such as naloxone (Narcan) promptly reverse the central nervous system (CNS)depressant effects of opioids. In addition, the antagonist counters the effect of the stress-induced levels of endorphins. An opioid antagonist is especially valuable if labor is more rapid than expected, and birth is anticipated when the opioid is at its peak effect. The woman should be told that the pain that was relieved by the opioid analgesic will return with administration of the opioid antagonist. The woman should be told that the pain that was relieved by the opioid analgesic will return with administration of the opioid antagonist. Her pain level will increase rather than decrease. Opioid antagonists have no effect on anxiety levels. They are given primarily to reverse excessive CNS depression in the mother, newborn, or both. An opioid antagonist (e.g., naloxone) has no effect on the mothers urge or ability to push. The practice of giving lower doses of IV opioids has reduced the incidence and severity of opioid-induced CNS depression; therefore, opioid antagonists are used less frequently.

With regard to the procedures and criteria for admitting a woman to the hospital labor unit, nurses should be aware that: a. She is not considered to be in true labor (according to the Emergency Medical Treatment and Active Labor Act [EMTALA]) until a qualified health care provider says she is b. She can have only her male partner or predesignated doula with her at assessment c. Her weight gain is calculated to determine whether she is at greater risk for complications d. The nurse should listen politely to the womans previous birthing experiences but should keep in mind that each birth is a unique experience

Her weight gain is calculated to determine whether she is at greater risk for complications Increasing numbers of women are overweight or obese. Excessive size can make nursing care during labor and birth more difficult and places the woman at risk for complications such as operative birth, infection, and blood clots. According to EMTALA, a woman is entitled to active labor care and is presumed to be in true labor until a qualified health care provider certifies otherwise. A woman may have anyone she wishes present for her support. The details of previous birthing experiences are important. Not only the mechanics of labor and the outcome but also the womans perceptions can influence her present attitude.

The nurse expects which maternal cardiovascular finding during labor? a. Increased cardiac output b. Decreased pulse rate c. Decreased white blood cell (WBC) count d. Decreased blood pressure

Increased cardiac output During each contraction, 400 ml of blood is emptied from the uterus into the maternal vascular system. This increases cardiac output by about 10% to 15% in the first stage of labor and by about 30% to 50% in the second stage. The heart rate increases slightly during labor. The WBC count can increase during labor. During the first stage of labor, uterine contractions cause systolic readings to increase by about 10 mm Hg. During the second stage, contractions may cause systolic pressures to increase by 30 mm Hg and diastolic readings to increase by 25 mm Hg.

According to professional standards (AWHONN, 2007) the nonanesthetist registered nurse caring for a woman with an epidural is permitted to perform all actions except: a. Monitor the status of the woman and fetus b. Initiate epidural anesthesia c. Replace empty infusion bags with the same medication and concentrate d. Stop the infusion and initiate emergency measures

Initiate epidural anesthesia Only qualified, licensed anesthesia care providers are permitted to insert a catheter, initiate epidural anesthesia, verify catheter placement, inject medication through the catheter, or alter the medication or medications including type, amount, or rate of infusion. The nonanesthetist nurse is permitted to monitor the status of the woman, the fetus, and the progress of labor. Replacement of the empty infusion bags or syringes with the same medication and concentration is permitted. If the need arises the nurse may stop the infusion, initiate emergency measures, and remove the catheter if properly educated to do so. Complications can require immediate interventions. Nurses must be prepared to provide safe and effective care during the emergency situation.

When planning care for a laboring woman whose membranes have ruptured, the nurse recognizes that the womans risk for _____ has increased. a. Intrauterine infection b. Hemorrhage c. Precipitous labor d. Supine hypotension

Intrauterine infection When the membranes rupture, microorganisms from the vagina can ascend into the amniotic sac, causing chorioamnionitis and placentitis. Rupture of membranes (ROM) is not associated with fetal or maternal bleeding. Although ROM may increase the intensity of contractions and facilitate active labor, it does not result in precipitous labor.

With regard to the process of augmentation of labor, a nurse should be aware that it: a. Is part of the active management of labor that is instituted when the labor process is unsatisfactory b. Relies on more invasive methods when oxytocin and amniotomy have failed c. Is a modern management term to cover up the negative connotations of forceps-assisted birth d. Uses vacuum cups

Is part of the active management of labor that is instituted when the labor process is unsatisfactory Augmentation is part of the active management of labor that stimulates uterine contractions after labor has started but is not progressing satisfactorily. Augmentation uses amniotomy and oxytocin infusion, as well as some gentler, noninvasive methods. Forceps-assisted births are less common than in the past and not considered a method of augmentation. A vacuum assist occurs during birth if the mother is too exhausted to push. Vacuum extraction is not considered an augmentation methodology.

Nurses should be aware that the induction of labor: a. Can be achieved by external and internal version techniques b. Is also known as a trial of labor (TOL) c. Is almost always done for medical reasons d. Is rated for viability by a Bishop score

Is rated for viability by a Bishop score Induction of labor is likely to be more successful with a Bishop score of 9 or higher for first-time mothers, 5 or higher for veterans. Version is turning of the fetus to a better position by a physician for an easier or safer birth. A trial of labor is the observance of a woman and her fetus for several hours of active labor to assess the safety of vaginal birth. Two thirds of cases of induced labor are elective and not done for medical reasons.

The most critical nursing action in caring for the newborn immediately after birth is: a. Keeping the airway clear b. Fostering parent-newborn attachment c. Drying the newborn and wrapping the infant in a blanket d. Administering eye drops and vitamin K

Keeping the airway clear The care given immediately after the birth focuses on assessing and stabilizing the newborn. Although fostering parent-newborn attachment is an important task for the nurse, it is not the most critical nursing action in caring for the newborn immediately after birth. The care given immediately after birth focuses on assessing and stabilizing the newborn. The nursing activities are (in order of importance) to maintain a patent airway, support respiratory effort, and prevent cold stress by drying the newborn and covering with a warmed blanket or placing the newborn under a radiant warmer. After the newborn has been stabilized, the nurse assesses the newborns physical condition, weighs and measures the newborn, administers prophylactic eye ointment and a vitamin K injection, affixes an identification bracelet, wraps the newborn in warm blankets, and then gives the partner or the mother the infant.

With regard to the process of inducing labor, nurses should be aware that: a. Ripening the cervix usually results in a decreased success rate for induction b. Labor sometimes can be induced with balloon catheters or laminaria tents c. Oxytocin is less expensive than prostaglandins and more effective but creates greater health risks d. Amniotomy can be used to make the cervix more favorable for labor

Labor sometimes can be induced with balloon catheters or laminaria tents These are mechanical means of ripening the cervix. Ripening the cervix, making it softer and thinner, increases the success rate of induced labor. Prostaglandin E1 is less expensive and more effective than oxytocin but carries a greater risk. Amniotomy is the artificial rupture of membranes, which is used to induce labor only when the cervix is already ripe.

Which fetal heart rate (FHR) finding concerns the nurse during labor? a. Accelerations with fetal movement b. Early decelerations c. An average FHR of 126 beats/min d. Late decelerations

Late decelerations Late decelerations are caused by uteroplacental insufficiency and are associated with fetal hypoxemia. They are considered ominous if persistent and uncorrected. Accelerations in the FHR are an indication of fetal well-being. Early decelerations in the FHR are associated with head compression as the fetus descends into the maternal pelvic outlet; they generally are not a concern during normal labor. An FHR finding of 126 beats/min is normal and not a concern.

Which statement correctly describes the effects of various pain factors? a. Higher prostaglandin levels arising from dysmenorrhea can blunt the pain of childbirth. b. Upright positions in labor increase the pain factor because they cause greater fatigue. c. Women who move around trying different positions are experiencing more pain. d. Levels of pain-mitigating beta-endorphins are higher during a spontaneous, natural childbirth.

Levels of pain-mitigating beta-endorphins are higher during a spontaneous, natural childbirth. Higher endorphin levels help women tolerate pain and reduce anxiety and irritability. Higher prostaglandin levels correspond to more severe labor pains. Upright positions in labor usually result in improved comfort and less pain. Moving freely to find more comfortable positions is important for reducing pain and muscle tension.

Signs that precede labor include (choose all that apply): a. Lightening b. Exhaustion c. Bloody show d. Rupture of membranes e. Decreased fetal movement

Lightening Bloody show Rupture of membranes Signs that precede labor may include lightening, urinary frequency, backache, weight loss, surge of energy, bloody show, and rupture of membranes. Many women experience a burst of energy before labor. A decrease in fetal movement is an ominous sign that does not always correlate with labor.

A woman who has a history of sexual abuse may have a number of traumatic memories triggered during labor. She may fight the labor process and react with pain or anger. Alternately, she may become a passive player and emotionally absent herself from the process. The nurse is in a unique position of being able to assist the client to associate the sensations of labor with the process of childbirth and not the past abuse. The nurse can implement a number of care measures to help her client view the childbirth experience in a positive manner. Which intervention is key for the nurse to use while providing care? a. Tell the client to relax and that it wont hurt much b. Limit the number of procedures that invade her body c. Reassure the client that as the nurse you know what is best d. Allow unlimited care providers to be with the client

Limit the number of procedures that invade her body The number of invasive procedures such as vaginal examinations, internal monitoring, and IV therapy should be limited as much as possible. The nurse should always avoid words and phrases that may result in the clients recalling the phrases of her abuser (i.e., Relax, this wont hurt or Just open your legs). The womans sense of control should be maintained at all times. The nurse should explain procedures at the clients pace and wait for permission to proceed. Protecting the clients environment by providing privacy and limiting the number of staff who observe the client will help to make her feel safe.

What position is least effective when gravity is desired to assist in fetal descent? a. Lithotomy b. Kneeling c. Sitting d. Walking

Lithotomy The predominant position in the United States for physician-attended births is the lithotomy position, which requires a woman to be in a reclined position with her legs in stirrups. Gravity has little effect in this position. Kneeling helps align the fetus within the pelvic outlet and allows gravity to assist in fetal descent. Sitting helps align the fetus within the pelvic outlet and allows gravity to assist in fetal descent. Walking helps align the fetus within the pelvic outlet and allows gravity to assist in fetal descent.

Nursing care measures are commonly offered to women in labor. Which nursing measure reflects application of the gate-control theory? a. Massage the womans back. b. Change the womans position. c. Give the prescribed medication. d. Encourage the woman to rest between contractions.

Massage the womans back. According to the gate-control theory, pain sensations travel along sensory nerve pathways to the brain, but only a limited number of sensations, or messages, can travel through these nerve pathways at one time. Distraction techniques, such as massage or stroking, music, focal points, and imagery, reduce or completely block the capacity of nerve pathways to transmit pain. These distractions are thought to work by closing down a hypothetic gate in the spinal cord, thus preventing pain signals from reaching the brain. The perception of pain is thereby diminished. Changing the womans position does not reduce or block the capacity of nerve pathways to transmit pain using the gate-control theory. Pain medication does not reduce or block the capacity of nerve pathways to transmit pain using the gate-control theory. Resting between contractions does not reduce or block the capacity of nerve pathways to transmit pain using the gate-control theory.

As the United States and Canada continue to become more culturally diverse, it is increasingly important for the nursing staff to recognize a wide range of varying cultural beliefs and practices. Nurses need to develop respect for these practices and learn to incorporate them into a mutually agreed-upon plan of care. Although it is common practice in the United States for the father of the baby to be present at the birth, in many societies this is not the case. When implementing care, the nurse should anticipate that a woman from which country will have the babys father in attendance? a. Mexico b. China c. Iran d. India

Mexico A woman from Mexico may be stoic about discomfort until the second stage, at which time she will request pain relief. Fathers and female relatives are usually in attendance during the second stage of labor. The father of the baby is expected to provide encouragement, support, and reassurance that all will be well. In China fathers are usually not present. The side-lying position is preferred for labor and birth because it is believed that this will reduce trauma to the infant. The client has a stoic response to pain. In Iran the father will not be present. Female support persons and female health care providers are preferred. For many, a male caregiver is unacceptable. In India the father is usually not present, but female relatives are usually present. Natural childbirth methods are preferred.

The slight overlapping of cranial bones or shaping of the fetal head during labor is called: a. Lightening b. Molding c. Ferguson reflex d. Valsalva maneuver

Molding Molding also permits adaptation to various diameters of the maternal pelvis. Lightening is the mothers sensation of decreased abdominal distention, which usually occurs the week before labor. The Ferguson reflex is the contraction urge of the uterus after stimulation of the cervix. The Valsalva maneuver describes conscious pushing during the second stage of labor.

The nurse providing care for the laboring woman understands that variable fetal heart rate (FHR) decelerations are caused by: a. Altered fetal cerebral blood flow b. Umbilical cord compression c. Uteroplacental insufficiency d. Fetal hypoxemia

Umbilical cord compression Variable decelerations can occur any time during the uterine contracting phase and are caused by compression of the umbilical cord. Altered fetal cerebral blood flow results in early decelerations in the FHR. Uteroplacental insufficiency results in late decelerations in the FHR. Fetal hypoxemia results in tachycardia initially, then bradycardia if hypoxia continues.

With regard to nerve block analgesia and anesthesia, nurses should be aware that: a. Most local agents are related chemically to cocaine and end in the suffix caine b. Local perineal infiltration anesthesia is effective when epinephrine is added, but it can be injected only once c. A pudendal nerve block is designed to relieve the pain from uterine contractions d. A pudendal nerve block, if done correctly, does not significantly lessen the bearing-down reflex

Most local agents are related chemically to cocaine and end in the suffix caine Common agents include lidocaine and chloroprocaine. Injections can be repeated to prolong the anesthesia. A pudendal nerve block relieves pain in the vagina, vulva, and perineum but not the pain from uterine contractions. A pudendal nerve block lessens or shuts down the bearing-down reflex.

A laboring woman received meperidine (Demerol) IV 90 minutes before she gave birth. Which medication should be available to reduce the postnatal effects of meperidine on the neonate? a. Fentanyl (Sublimaze) b. Promethazine (Phenergan) c. Naloxone (Narcan) d. Nalbuphine (Nubain)

Naloxone (Narcan) An opioid antagonist can be given to the newborn as one part of the treatment for neonatal narcosis, which is a state of central nervous system (CNS) depression in the newborn produced by an opioid. Opioid antagonists such as naloxone can promptly reverse the CNS depressant effects, especially respiratory depression. Fentanyl (Sublimaze) does not act as opioid antagonists to reduce the postnatal effects of meperidine on the neonate. Promethazine (Phenergan) does not act as opioid antagonists to reduce the postnatal effects of meperidine on the neonate. Nalbuphine (Nubain) does not act as opioid antagonists to reduce the postnatal effects of meperidine on the neonate.

Maternal hypotension is a potential side effect of regional anesthesia and analgesia. What nursing interventions could you use to raise the clients blood pressure? Choose all that apply. a. Place the woman in a supine position. b. Place the woman in a lateral position. c. Increase IV fluids. d. Administer oxygen. e. Perform a vaginal examination.

Place the woman in a lateral position. Increase IV fluids Administer oxygen Nursing interventions for maternal hypotension arising from analgesia or anesthesia include turning the woman to a lateral position, increasing IV fluids, administering oxygen via face mask, elevating the womans legs, notifying the physician, administering an IV vasopressor, and monitoring the maternal and fetal status at least every 5 minutes until these are stable. Placing the client in a supine position causes venous compression, thereby limiting blood flow to and oxygenation of the placenta and fetus. A sterile vaginal examination has no bearing on maternal blood pressure.

The factors that affect the process of labor and birth, known commonly as the five Ps, include all except: a. Passenger b. Passageway c. Powers d. Pressure

Pressure The five Ps are passenger (fetus and placenta), passageway (birth canal), powers (contractions), position of the mother, and psychologic response. Pressure is not one of the five Ps. The passenger (fetus and placenta) is one of the five Ps. The passageway (birth canal) is also one of the five Ps. Powers (contractions) are a necessary component of the five Ps.

Nurses should know some basic definitions concerning preterm birth, preterm labor, and low birth weight. For instance: a. The terms preterm birth and low birth weight can be used interchangeably b. Preterm labor is defined as cervical changes and uterine contractions occurring between 20 and 37 weeks of pregnancy c. Low birth weight is anything below 3.7 lb d. In the United States early in this century, preterm birth accounted for 18% to 20% of all births

Preterm labor is defined as cervical changes and uterine contractions occurring between 20 and 37 weeks of pregnancy Before 20 weeks it is not viable (miscarriage); after 37 weeks, it can be considered term. Although these terms are used interchangeably, they have different meanings: preterm birth describes the length of gestation (before 37 weeks) regardless of weight; low birth weight describes weight only (2500 g or less) at the time of birth, whenever it occurs. Low birth weight is anything below 2500 g, or about 5.5 pounds. In 2006, the preterm birth rate in the United States was 12.8%; however, the rate is increasing in frequency.

In planning for home care of a woman with preterm labor, the nurse needs to address which concern? a. Nursing assessments are different from those done in the hospital setting. b. Restricted activity and medications are necessary to prevent recurrence of preterm labor. c. Prolonged bed rest may cause negative physiologic effects. d. Home health care providers are necessary.

Prolonged bed rest may cause negative physiologic effects. Prolonged bed rest may cause adverse effects such as weight loss, loss of appetite, muscle wasting, weakness, bone demineralization, decreased cardiac output, risk for thrombophlebitis, alteration in bowel functions, sleep disturbance, and prolonged postpartum recovery. Nursing assessments differ somewhat from those performed in the acute care setting, but this is not the concern that needs to be addressed. Restricted activity and medications may prevent preterm labor but not in all women. Additionally, the plan of care is individualized to meet the needs of each client. Many women receive home health nurse visits, but care is individualized for each woman.

What three measures should the nurse implement to provide intrauterine resuscitation? Select the best response that indicates the priority of actions that should be taken, starting with the most important. a. Call the provider, reposition the mother, and perform a vaginal exam. b. Provide oxygen via face mask, reposition the mother, and increase IV fluid. c. Administer oxygen to the mother, increase IV fluid, and notify the health care provider. d. Perform a vaginal examination, reposition the mother, and provide oxygen via face mask.

Provide oxygen via face mask, reposition the mother, and increase IV fluid. Basic interventions for management of any abnormal fetal heart rate pattern includes administer oxygen by nonrebreather face mask at a rate of 8 to 10 L/min, assist the woman to a side-lying (lateral) position, and increase blood volume by increasing the rate of the primary IV infusion. The purpose of these interventions is to improve uterine blood flow and intervillous space blood flow and increase maternal oxygenation and cardiac output. The term intrauterine resuscitation is sometimes used to refer to these interventions. Basic corrective measures include providing O2, instituting maternal position changes, and increasing IV fluid volume. If these interventions do not resolve the fetal heart rate issue quickly, the primary provider should be notified immediately. In this scenario the nurse failed to alter the womans position. To improve uterine blood flow, the woman should be repositioned onto her side. If these interventions do not resolve the fetal heart rate issue quickly, the primary provider should be notified immediately. Performing a vaginal examination would not be helpful at this time. In this scenario the nurse should have begun by applying O2 at 8 to 10 L/min by nonrebreather face mask.

When assessing the fetus using Leopold maneuvers, the nurse feels a round, firm, movable fetal part in the fundal portion of the uterus and a long, smooth surface in the mothers right side close to midline. What is the likely position of the fetus? a. ROA b. LSP c. RSA d. LOA

RSA Fetal position is denoted with a three-letter abbreviation. The first letter indicates the presenting part in either the right or left side of the maternal pelvis. The second letter indicates the anatomic presenting part of the fetus. The third letter stands for the location of the presenting part in relation to the anterior, posterior, or transverse portion of the maternal pelvis. Palpation of a round, firm fetal part in the fundal portion of the uterus would be the fetal head, indicating that the fetus is in a breech position with the sacrum as the presenting part in the maternal pelvis. Palpation of the fetal spine along the mothers right side denotes the location of the presenting part in the mothers pelvis. The ability to palpate the fetal spine indicates that the fetus is anteriorly positioned in the maternal pelvis. This fetus is positioned anteriorly in the right side of the maternal pelvis with the sacrum as the presenting part. RSA is the correct three-letter abbreviation to indicate this fetal position.

An 18-year-old pregnant woman, gravida 1, is admitted to the labor and birth unit with moderate contractions every 5 minutes that last 40 seconds. The woman states, My contractions are so strong, I dont know what to do. The nurse should: a. Assess for fetal well-being b. Encourage the woman to lie on her side c. Disturb the woman as little as possible d. Recognize that pain is personalized for each individual

Recognize that pain is personalized for each individual Each womans pain during childbirth is unique and is influenced by a variety of physiologic, psychosocial, and environmental factors. A critical issue for the nurse is how support can make a difference in the pain of the woman during labor and birth. This scenario includes no information that would indicate fetal distress or a logical reason to be overly concerned about the well-being of the fetus. The left lateral position is used to alleviate fetal distress, not maternal stress. The nurse has an obligation to provide physical, emotional, and psychosocial care and support to the laboring woman. This client clearly needs support.

Prostaglandin gel has been ordered for a pregnant woman at 43 weeks of gestation. The nurse recognizes that this medication is administered to: a. Enhance uteroplacental perfusion in an aging placenta b. Increase amniotic fluid volume c. Ripen the cervix in preparation for labor induction d. Stimulate the amniotic membranes to rupture

Ripen the cervix in preparation for labor induction Preparations of prostaglandin E1 and E2 are effective when used before induction to ripen (soften and thin) the cervix. Uteroplacental perfusion is not altered by the use of prostaglandins. The insertion of prostaglandin gel has no effect on the level of amniotic fluid. In some cases, women will spontaneously begin laboring after the administration of prostaglandins, thereby eliminating the need for oxytocin. It is not common for a woman to rupture her membranes as a result of prostaglandin use.

A woman is having her first child. She has been in labor for 15 hours. Two hours ago her vaginal examination revealed the cervix to be dilated to 5 cm and 100% effaced, and the presenting part was at station 0. Five minutes ago her vaginal examination indicated that there had been no change. What abnormal labor pattern is associated with this description? a. Prolonged latent phase b. Protracted active phase c. Secondary arrest d. Protracted descent

Secondary arrest With a secondary arrest of the active phase, the progress of labor has stopped. This client has not had any anticipated cervical change, indicating an arrest of labor. In the nulliparous woman, a prolonged latent phase typically lasts more than 20 hours. A protracted active phase, the first or second stage of labor, is prolonged (slow dilation). With protracted descent, the fetus fails to descend at an anticipated rate during the deceleration phase and second stage of labor.

Which factors influence cervical dilation? Choose all that apply. a. Strong uterine contractions b. The force of the presenting fetal part against the cervix c. The size of the female d. The pressure applied by the amniotic sac e. Scarring of the cervix

Strong uterine contractions The force of the presenting fetal part against the cervix The pressure applied by the amniotic sac Scarring of the cervix Dilation of the cervix occurs by the drawing upward of the musculofibrous components of the cervix, which is caused by strong uterine contractions. Pressure exerted by the amniotic fluid while the membranes are intact or by the force applied by the presenting part also can promote cervical dilation. Scarring of the cervix as a result of a previous infection or surgery may slow cervical dilation.

A first-time mother is concerned about the type of medications she will receive during labor. She is in a fair amount of pain and is nauseated. Additionally she appears to be very anxious. You explain that opioid analgesics often are used with sedatives because: a. The two together work the best for you and your baby. b. Sedatives help the opioid work better, and they will help relax you and relieve your nausea. c. They work better together so you can sleep until you have the baby. d. This is what the doctor has ordered for you.

Sedatives help the opioid work better, and they will help relax you and relieve your nausea. Sedatives may be used to reduce the nausea and vomiting that often accompany opioid use. Additionally, some ataractics reduce anxiety and apprehension and potentiate the opioid analgesic affects. A potentiator may cause two drugs to work together more effectively, but it does not ensure maternal or fetal complications. Sedation may be a related effect of some ataractics, but it is not the goal. Furthermore, a woman is unlikely to be able to sleep through transitional labor and birth. Although the doctor may have ordered the medications, it is not an acceptable comment for the nurse to make.

In evaluating the effectiveness of magnesium sulfate for the treatment of preterm labor, which finding alerts the nurse to possible side effects? a. Urine output of 160 ml in 4 hours b. Deep tendon reflexes 2+ and no clonus c. Respiratory rate (RR) of 16 breaths/min d. Serum magnesium level of 10 mg/dl

Serum magnesium level of 10 mg/dl The therapeutic range for magnesium sulfate management is 5 to 8 mg/dl. A serum magnesium level of 10 mg/dl could lead to signs and symptoms of magnesium toxicity, including oliguria and respiratory distress. Urine output of 160 ml in 4 hours is a normal finding. Deep tendon reflexes of 2+ is a normal finding. RR of 16 breaths/min is a normal finding.

With regard to a pregnant womans anxiety and pain experience, nurses should be aware that: a. Even mild anxiety must be treated b. Severe anxiety increases tension, which increases pain, which in turn increases fear and anxiety, and so on c. Anxiety may increase the perception of pain, but it does not affect the mechanism of labor d. Women who have had a painful labor will have learned from the experience and have less anxiety the second time because of increased familiarity

Severe anxiety increases tension, which increases pain, which in turn increases fear and anxiety, and so on Anxiety and pain reinforce each other in a bad cycle. Mild anxiety is normal for a woman in labor and likely needs no special treatment other than the standard reassurances. Anxiety increases muscle tension and ultimately can build sufficiently to slow the progress of labor. Unfortunately, an anxious, painful first labor is likely to carry over, through expectations and memories, into an anxious and painful experience in the second pregnancy.

A primigravida at 40 weeks of gestation is having uterine contractions every 1 to 2 minutes and says that they are very painful. Her cervix is dilated 2 cm and has not changed in 3 hours. The woman is crying and wants an epidural. What is the likely status of this womans labor? a. She is exhibiting hypotonic uterine dysfunction. b. She is experiencing a normal latent stage. c. She is exhibiting hypertonic uterine dysfunction. d. She is experiencing pelvic dystocia.

She is exhibiting hypertonic uterine dysfunction. Women who experience hypertonic uterine dysfunction, or primary dysfunctional labor, often are anxious first-time mothers who are having painful and frequent contractions that are ineffective at causing cervical dilation or effacement to progress. With hypotonic uterine dysfunction, the woman initially makes normal progress into the active stage of labor and then the contractions become weak and inefficient or stop altogether. The contraction pattern seen in this woman signifies hypertonic uterine activity. Typically uterine activity in this phase occurs at 4- to 5-minute intervals lasting 30 to 45 seconds. Pelvic dystocia can occur whenever contractures of the pelvic diameters reduce the capacity of the bony pelvis, including the inlet, midpelvis, outlet, or any combination of these planes.

Breathing patterns are taught to laboring women. Which breathing pattern would the nurse support for the woman and her coach during the latent phase of the first stage of labor if the couple had attended Lamaze classes? a. Slow-paced breathing b. Deep abdominal breathing c. Modified-paced breathing d. Patterned-paced breathing

Slow-paced breathing Slow-paced breathing is approximately half the womans normal breathing rate. It is used during the early stages of labor when a woman can no longer walk or talk through her contractions. No such pattern as deep abdominal breathing exists in Lamaze. Modified-paced breathing is shallow breathing that is twice the womans normal breathing rate. It is used when labor progresses and the woman can no longer maintain relaxation through paced breathing. Patterned-pace breathing is a fast, 4:1 breathe, breathe, breathe, blow pattern that is used during the transitional phase of labor just before pushing and delivery.

A woman in preterm labor at 30 weeks of gestation receives two 12-mg doses of betamethasone intramuscularly. The purpose of this pharmacologic treatment is to: a. Stimulate fetal surfactant production b. Reduce maternal and fetal tachycardia associated with ritodrine administration c. Suppress uterine contractions d. Maintain adequate maternal respiratory effort and ventilation during magnesium sulfate therapy

Stimulate fetal surfactant production Antenatal glucocorticoids given as IM injections to the mother accelerate fetal lung maturity. Propranolol (Inderal) is given to reduce the effects of ritodrine administration. Betamethasone has no effect on uterine contractions. Calcium gluconate is given to reverse the respiratory depressive effects of magnesium sulfate therapy.

The nurse expects to administer an oxytocic (e.g., Pitocin, Methergine) to a woman after expulsion of her placenta to: a. Relieve pain b. Stimulate uterine contraction c. Prevent infection d. Facilitate rest and relaxation

Stimulate uterine contraction Oxytocics stimulate uterine contractions, which reduce blood loss after the third stage of labor.

A pregnant woman is at 38 weeks of gestation. She wants to know if any signs indicate labor is getting closer to starting. The nurse informs the woman that which of the following is a sign that labor may begin soon? a. Weight gain of 1.5 to 2 kg (3 to 4 lb) b. Increase in fundal height c. Urinary retention d. Surge of energy

Surge of energy Women speak of having a burst of energy before labor. The woman may lose 0.5 to 1.5 kg, the result of water loss caused by electrolyte shifts, which in turn are caused by changes in the estrogen and progesterone levels. When the fetus descends into the true pelvis (called lightening), the fundal height may decrease.

According to the National Institute of Child Health and Human Development (NICHD) Three-Tier System of Fetal Heart Rate Classification, category III tracings include all fetal heart rate (FHR) tracings not categorized as category I or II. Which characteristics of the fetal heart rate belong in category III? a. Baseline rate of 110 to 160 beats/min b. Tachycardia c. Absent baseline variability not accompanied by recurrent decelerations d. Variable decelerations with other characteristics such as shoulders or overshoots e. Absent baseline variability with recurrent variable decelerations f. Bradycardia

Tachycardia Variable decelerations with other characteristics such as shoulders or overshoots Absent baseline variability with recurrent variable decelerations Bradycardia Tachycardia, variable decelerations with other characteristics, absent baseline variability with recurrent variable decelerations, and bradycardia are characteristics that are considered nonreassuring or abnormal and belong in category III. A fetal heart rate of 110 to 160 beats/min is considered normal and belongs in category I. Absent baseline variability not accompanied by recurrent decelerations is a category II characteristic.

A means of controlling the birth of the fetal head with a vertex presentation is: a. The Ritgen maneuver b. Fundal pressure c. The lithotomy position d. The De Lee apparatus

The Ritgen maneuver The Ritgen maneuver extends the head during the actual birth and protects the perineum. Gentle, steady pressure against the fundus of the uterus facilitates vaginal birth. The lithotomy position has been commonly used in Western cultures, partly because it is convenient for the health care provider. The De Lee apparatus is used to suction fluid from the infants mouth.

The nurse has received a report about a woman in labor. The womans last vaginal examination was recorded as 3 cm, 30%, and 2. The nurses interpretation of this assessment is that: a. The cervix is effaced 3 cm, it is dilated 30%, and the presenting part is 2 cm above the ischial spines b. The cervix is 3 cm dilated, it is effaced 30%, and the presenting part is 2 cm above the ischial spines c. The cervix is effaced 3 cm, it is dilated 30%, and the presenting part is 2 cm below the ischial spines d. The cervix is dilated 3 cm, it is effaced 30%, and the presenting part is 2 cm below the ischial spines

The cervix is 3 cm dilated, it is effaced 30%, and the presenting part is 2 cm above the ischial spines

The nurse recognizes that a woman is in true labor when she states: a. I passed some thick, pink mucus when I urinated this morning. b. My bag of waters just broke. c. The contractions in my uterus are getting stronger and closer together. d. My baby dropped, and I have to urinate more frequently now.

The contractions in my uterus are getting stronger and closer together. Regular, strong contractions with the presence of cervical change indicate that the woman is experiencing true labor. Loss of the mucous plug (operculum) often occurs during the first stage of labor or before the onset of labor, but it is not the indicator of true labor. Spontaneous rupture of membranes often occurs during the first stage of labor, but it is not the indicator of true labor. The presenting part of the fetus typically becomes engaged in the pelvis at the onset of labor, but this is not the indicator of true labor.

With regard to the care management of preterm labor, nurses should be aware that: a. Because all women must be considered at risk for preterm labor and prediction is so hit and miss, teaching pregnant women the symptoms probably causes more harm through false alarms b. Braxton Hicks contractions often signal the onset of preterm labor c. Because preterm labor is likely to be the start of an extended labor, a woman with symptoms can wait several hours before contacting the primary caregiver d. The diagnosis of preterm labor is based on gestational age, uterine activity, and progressive cervical change

The diagnosis of preterm labor is based on gestational age, uterine activity, and progressive cervical change Gestational age of 20 to 37 weeks, uterine contractions, and a cervix that is 80% effaced or dilated 2 cm indicates preterm labor. It is essential that nurses teach women how to detect the early symptoms of preterm labor. Braxton Hicks contractions resemble preterm labor contractions, but they are not true labor. Waiting too long to see a health care provider could result in essential medications failing to be administered. Preterm labor is not necessarily long-term labor.

With regard to the turns and other adjustments of the fetus during the birth process, known as the mechanism of labor, nurses should be aware that: a. The seven critical movements must progress in a more or less orderly sequence b. Asynclitism sometimes is achieved by means of the Leopold maneuver c. The effects of the forces determining descent are modified by the shape of the womans pelvis and the size of the fetal head d. At birth the baby is said to achieve restitution; that is, a return to the C-shape of the womb

The effects of the forces determining descent are modified by the shape of the womans pelvis and the size of the fetal head The size of the maternal pelvis and the ability of the fetal head to mold also affect the process. The seven identifiable movements of the mechanism of labor occur in combinations simultaneously, not in precise sequences. Asynclitism is the deflection of the babys head; the Leopold maneuver is a means of judging descent by palpating the mothers abdomen. Restitution is the rotation of the babys head after the infant is born.

As part of the physical examination component of assessment, Leopold maneuvers are used to help identify all of the following except: a. The gender of the fetus b. The number of fetuses c. The fetal lie and attitude d. The degree of the presenting parts descent into the pelvis

The gender of the fetus Leopold maneuvers help identify the number of fetuses, the fetal lie and attitude, and the presenting parts degree of descent into the pelvis. The gender of the fetus cannot be determined by performing Leopold maneuvers. Leopold maneuvers help identify the number of fetuses. Leopold maneuvers assist the nurse in determining the fetal lie and attitude. Leopold maneuvers can be used to ascertain the presenting parts degree of descent into the pelvis.

A nurse should be aware that an effective plan to achieve adequate pain relief without maternal risk is most effective if: a. The mother gives birth without any analgesic or anesthetic b. The mothers priorities and preferences are incorporated into the plan c. The primary health care provider decides the best pain relief for the mother and family d. The nurse informs the family of all alternative methods of pain relief available in the hospital setting

The mothers priorities and preferences are incorporated into the plan Current evidence indicates that a womans satisfaction with her labor and birth experience is determined by how well her personal expectations of childbirth are met and the quality of support and interaction she receives from her caregivers. The assessment of the woman, her fetus, and her labor is a joint effort of the nurse and the primary health care providers, who consult the woman about their findings and recommendations. The needs of each woman are different, and many factors must be considered before a decision is made whether pharmacologic methods, nonpharmacologic methods, or a combination of the two will be used to manage labor pain. Although the predominant medical approach to labor is that it is painful, and the pain must be removed, an alternative view is that labor is a natural process, and women can experience comfort and transcend the discomfort or pain to reach the joyful outcome of birth. The provider should collaborate with the mother and her support persons as to the ideal plan to achieve adequate pain relief. The most helpful interventions in enhancing comfort are a caring nursing approach and a supportive presence. Discussing options for nonpharmacologic methods of pain relief is also helpful. Ideally this is done prior to the onset of labor.

Regarding how the fetus moves through the birth canal, nurses should be aware that: a. The fetal attitude describes the angle at which the fetus exits the uterus b. Of the two primary fetal lies, the horizontal lie is that in which the long axis of the fetus is parallel to the long axis of the mother c. The normal attitude of the fetus is called general flexion d. The transverse lie is preferred for vaginal birth

The normal attitude of the fetus is called general flexion The normal attitude of the fetus is general flexion. The fetal attitude is the relation of fetal body parts to each other. The normal attitude is called general flexion. The horizontal lie is perpendicular to the mother; in the longitudinal (or vertical) lie, the long axes of the fetus and the mother are parallel. Vaginal birth cannot occur if the fetus stays in a transverse lie.

Nurses alert to signs of the onset of the second stage of labor can be certain that stage has begun when: a. The woman has a sudden episode of vomiting b. The nurse is unable to feel the cervix during a vaginal examination c. Bloody show increases d. The woman involuntarily tries to bear down

The nurse is unable to feel the cervix during a vaginal examination The only certain objective sign that the second stage has begun is the inability to feel the cervix because it is fully dilated and effaced. Sudden vomiting is a suggestion of second-stage labor; however, the only certain objective sign that the second stage has begun is the inability to feel the cervix because it is fully dilated and effaced. Bloody show increases during the second stage of labor; however, the inability to feel the cervix is an objective sign that the second stage has begun. The urge to bear down is a suggestion of second-stage labor. The only certain objective sign that the second stage has begun is the inability to feel the cervix because it is fully dilated and effaced.

The nurse should assist the laboring woman into a hands-and-knees position when: a. The occiput of the fetus is in a posterior position b. The fetus is at or above the ischial spines c. The fetus is in a vertex presentation d. The membranes rupture

The occiput of the fetus is in a posterior position The hands-and-knees position is effective in helping to rotate the fetus from a posterior to an anterior position. Many women experience the irresistible urge to push when the fetus is at the level of the ischial spines. In some cases this occurs before the woman is fully dilated. The woman should be instructed not to push until complete cervical dilation has occurred. No one position is correct for childbirth. The two most common positions assumed by women are the sitting and side-lying positions. The woman may be encouraged into a hands-and-knees position if the umbilical cord prolapsed when the membranes ruptured.

In order to help clients manage discomfort and pain during labor, nurses should be aware that: a. The predominant pain of the first stage of labor is the visceral pain located in the lower portion of the abdomen b. Referred pain is the extreme discomfort between contractions c. The somatic pain of the second stage of labor is more generalized and related to fatigue d. Pain during the third stage is a somewhat milder version of the second stage

The predominant pain of the first stage of labor is the visceral pain located in the lower portion of the abdomen This pain comes from cervical changes, distention of the lower uterine segment, and uterine ischemia. Referred pain occurs when the pain that originates in the uterus radiates to the abdominal wall, lumbosacral area of the back, iliac crests, and gluteal area. Second-stage labor pain is intense, sharp, burning, and localized. Third-stage labor pain is similar to that of the first stage.

What is an advantage of external electronic fetal monitoring? a. The ultrasound transducer can accurately measure short-term variability and beat-to-beat changes in the fetal heart rate (FHR). b. The tocotransducer can measure and record the frequency, regularity, intensity, and approximate duration of uterine contractions (UCs). c. The tocotransducer is especially valuable for measuring uterine activity (UA) during the first stage of labor. d. Once correctly applied by the nurse, the transducer need not be repositioned even when the woman changes positions.

The tocotransducer is especially valuable for measuring uterine activity (UA) during the first stage of labor. The tocotransducer is valuable for measuring uterine activity during the first stage of labor. This is especially true when the membranes are intact. Short-term variability and beat-to-beat changes cannot be measured with this technology. The tocotransducer cannot measure and record the intensity of UCs. The transducer must be repositioned when the woman or the fetus changes position.

A new client and her partner arrive on the labor, delivery, recovery, and postpartum (LDRP) unit for the birth of their first child. You apply the electronic fetal monitor to the woman. Her partner asks you to explain what is printing on the graph, referring to the EFM strip. He wants to know what the babys heart rate should be. Your best response is: a. Dont worry about that machine; thats my job. b. The top line graphs the babys heart rate. Generally the heart rate is between 110 and 160. The heart rate will fluctuate in response to what is happening during labor. c. The top line graphs the babys heart rate, and the bottom line lets me know how strong the contractions are. d. Your doctor will explain all of that later.

The top line graphs the babys heart rate. Generally the heart rate is between 110 and 160. The heart rate will fluctuate in response to what is happening during labor. Explaining what indicates a normal fetal heart rate educates the partner about fetal monitoring and provides support and information to alleviate his fears. Telling the partner not to worry discredits his feelings and does not provide the teaching he is requesting. Telling the partner that the graph indicates how strong the contractions are provides inaccurate information and does not address the partners concerns about the fetal heart rate. The fetal monitor graphs the frequency and duration of the contractions, not the intensity. Nurses should take every opportunity to provide client and family teaching, especially when information is requested.

With regard to a womans intake and output during labor, nurses should be aware that: a. The tradition of restricting the laboring woman to clear liquids and ice chips is being challenged because regional anesthesia is used more often than general anesthesia b. IV fluids usually are necessary to ensure that the laboring woman stays hydrated c. Routine use of an enema empties the rectum and is very helpful for producing a clean, clear delivery d. When a nulliparous woman experiences the urge to defecate, it often means birth will follow quickly

The tradition of restricting the laboring woman to clear liquids and ice chips is being challenged because regional anesthesia is used more often than general anesthesia Women are awake with regional anesthesia and are able to protect their own airway, which reduces the worry over aspiration. Routine IV fluids during labor are unlikely to be beneficial and may be harmful. Routine use of an enema is at best ineffective and may be harmful. Having the urge to defecate followed by the birth of her fetus is true for a multiparous woman but not for a first-timer.

Your client is in early labor, and you are discussing the pain relief options she is considering. She states that she wants an epidural no matter what! Your best response is: a. Ill make sure you get your epidural. b. You may only have an epidural if your doctor allows it. c. You may only have any epidural if you are going to deliver vaginally. d. The type of analgesia or anesthesia used is determined in part by the stage of your labor and the method of birth.

The type of analgesia or anesthesia used is determined in part by the stage of your labor and the method of birth. To avoid suppression of the progress of labor, pharmacologic measures for pain relief generally are not implemented until labor has advanced to the active phase of the first stage and the cervix is dilated approximately 4 to 5 cm. A plan of care is developed for each woman to address her particular clinical and nursing problems. The nurse collaborates with the primary health care provider and the laboring woman in selecting features of care relevant to the woman and her family. The decision whether to use an epidural to relieve labor pain is multifactorial. The nurse should not make a blanket statement guaranteeing the client one pharmacologic option over another until a complete history and physical examination has been obtained. A physicians order is required for pharmacologic options for pain management. However, this is not the nurses best response. An epidural is an effective pharmacologic pain management option for many laboring women. It can also be used for anesthesia control if the woman undergoes an operative delivery.

The nurse knows that the second stage of labor, the descent phase, has begun when: a. The amniotic membranes rupture b. The cervix cannot be felt during a vaginal examination c. The woman experiences a strong urge to bear down d. The presenting part is below the ischial spines

The woman experiences a strong urge to bear down During the descent phase of the second stage of labor, the woman may experience an increase in the urge to bear down. Rupture of membranes (ROM) has no significance in determining the stage of labor. The second stage of labor begins with full cervical dilation. Many women may have an urge to bear down when the presenting part is below the level of the ischial spines. This can occur during the first stage of labor, as early as 5 cm dilation.

A nurse might be called on to stimulate the fetal scalp: a. As part of fetal scalp blood sampling b. In response to tocolysis c. In preparation for fetal oxygen saturation monitoring d. To elicit an acceleration in the fetal heart rate (FHR)

To elicit an acceleration in the fetal heart rate (FHR) The scalp can be stimulated using digital pressure during a vaginal examination. Fetal scalp blood sampling involves swabbing the scalp with disinfectant before a sample is collected. The nurse stimulates the fetal scalp to elicit an acceleration of the FHR. Tocolysis is relaxation of the uterus. The nurse stimulates the fetal scalp to elicit an acceleration of the FHR. Fetal oxygen saturation monitoring involves the insertion of a sensor. The nurse stimulates the fetal scalp to elicit an acceleration of the FHR.

When using intermittent auscultation (IA) for a fetal heart rate (FHR), nurses should be aware that: a. They can be expected to cover only two or three clients when IA is the primary method of fetal assessment b. The best course is to use the descriptive terms associated with electronic fetal monitoring (EFM) when documenting results c. If the heartbeat cannot be found immediately, a shift must be made to electronic monitoring d. Ultrasound can be used to find the FHR and reassure the mother if initial difficulty was a factor

Ultrasound can be used to find the FHR and reassure the mother if initial difficulty was a factor Locating fetal heartbeats often takes time. Mothers can be reassured verbally and by the ultrasound pictures if that device is used to help locate the heartbeat. When used as the primary method of fetal assessment, IA requires a nurse-to-client ratio of one to one. Documentation should use only terms that can be numerically defined; the usual visual descriptions of EFM are inappropriate. Locating fetal heartbeats often takes time. Mothers can be reassured verbally and by the ultrasound pictures if that device is used to help locate the heartbeat.

A 25-year-old gravida 2, para 2-0-0-2 gave birth 4 hours ago to a 9-lb, 7-oz boy after augmentation of labor with oxytocin (Pitocin). She puts on her call light and asks for her nurse right away, stating Im bleeding a lot. The most likely cause of postpartum hemorrhage in this woman is: a. Retained placental fragments b. Unrepaired vaginal lacerations c. Uterine atony d. Puerperal infection

Uterine atony This woman gave birth to a macrosomic infant after oxytocin augmentation. The most likely cause of bleeding 4 hours after delivery, combined with these risk factors, is uterine atony. Although retained placental fragments may cause postpartum hemorrhage, this typically is detected in the first hour after delivery of the placenta and is not the most likely cause of hemorrhage in this woman. Although unrepaired vaginal lacerations may cause bleeding, they typically occur in the period immediately after birth. Puerperal infection can cause subinvolution and subsequent bleeding; however, this typically is detected after 24 hours postpartum.

The nurse recognizes that uterine hyperstimulation with oxytocin requires emergency interventions. What clinical cues alert the nurse that the woman is experiencing uterine hyperstimulation? Choose all that apply. a. Uterine contractions lasting <90 seconds and occurring >2 minutes in frequency b. Uterine contractions lasting >90 seconds and occurring <2 minutes in frequency c. Uterine tone <20 mm Hg d. Uterine tone >20 mm Hg e. Increased uterine activity accompanied by a nonreassuring fetal heart rate (FHR) and pattern

Uterine contractions lasting >90 seconds and occurring <2 minutes in frequency Uterine tone >20 mm Hg Increased uterine activity accompanied by a nonreassuring fetal heart rate (FHR) and pattern Uterine contractions that occur less than 2 minutes apart and last more than 90 seconds, a uterine tone of over 20 mm Hg, and a nonreassuring FHR and pattern are indications of uterine hyperstimulation with oxytocin administration. Uterine contractions that occur more than 2 minutes apart and last less than 90 seconds are the expected goal of oxytocin induction. A uterine tone of less than 20 mm Hg is normal.

A nurse caring for a woman in labor understands that maternal hypotension can result in: a. Early decelerations b. Fetal arrhythmias c. Uteroplacental insufficiency d. Spontaneous rupture of membranes

Uteroplacental insufficiency Low maternal blood pressure reduces placental blood flow during uterine contractions, resulting in fetal hypoxemia. Maternal hypotension does not result in early decelerations. Maternal hypotension is not associated with fetal arrhythmias. Spontaneous rupture of membranes is not a result of maternal hypotension.

The nurse providing care for a high risk laboring woman is alert for late fetal heart rate (FHR) decelerations. These late decelerations may be caused by: a. Altered cerebral blood flow b. Umbilical cord compression c. Uteroplacental insufficiency d. Meconium fluid

Uteroplacental insufficiency Uteroplacental insufficiency results in late decelerations in the FHR. Altered fetal cerebral blood flow results in early decelerations in the FHR. Umbilical cord compression results in variable decelerations in the FHR. Meconium-stained fluid may or may not produce changes in the fetal heart rate, depending on the gestational age of the fetus and whether other causative factors associated with fetal distress are present.

A nurse providing care for a laboring woman understands that amnioinfusion is used to treat: a. Variable decelerations b. Late decelerations c. Fetal bradycardia d. Fetal tachycardia

Variable decelerations Amnioinfusion is used during labor either to dilute meconium-stained amniotic fluid or to supplement the amount of amniotic fluid to reduce the severity of variable decelerations caused by cord compression. Late decelerations are unresponsive to amnioinfusion. Amnioinfusion is not appropriate for treatment of fetal bradycardia. Amnioinfusion has no bearing on fetal tachycardia.

The exact cause of preterm labor is unknown and believed to be multifactorial. Infection is thought to be a major factor in many preterm labors. Select the type of infection that has not been linked to preterm births: a. Viral b. Periodontal c. Cervical d. Urinary tract

Viral The infections that increase the risk of preterm labor and birth are bacterial. They include cervical, urinary tract, periodontal, and other bacterial infections. It is therefore important for the client to participate in early, continual, and comprehensive prenatal care. Recent evidence has shown a link between periodontal infections and preterm labor. Researchers recommend regular dental care before and during pregnancy, oral assessment as a routine part of prenatal care, and scrupulous oral hygiene in order to prevent infection. Cervical infections of a bacterial nature have been linked to preterm labor and birth. The presence of urinary tract infections increases the risk of preterm labor and birth.

A pregnant woman at 29 weeks of gestation has been diagnosed with preterm labor. Her labor is being controlled with tocolytic medications. She asks when she might be able to go home. What response by the nurse is most accurate? a. After the baby is born. b. When we can stabilize your preterm labor and arrange home health visits. c. Whenever the doctor says that it is okay. d. It depends on what kind of insurance coverage you have.

When we can stabilize your preterm labor and arrange home health visits. The clients preterm labor is being controlled with tocolytics. Once she is stable, home care may be a viable option for this type of client. Care of a woman with preterm labor is multifactorial; the goal is to prevent delivery. In many cases, this may be achieved at home. Care of the preterm client is multidisciplinary and multifactorial. Managed care may dictate earlier hospital discharges or a shift from hospital to home care. Insurance coverage may be one factor in client care, but ultimately, client safety remains the most important factor.


Ensembles d'études connexes

AWS Practitioner Exam: Knowledge Check Questions

View Set

Astronomy chapter 5 and chapter 6 homework

View Set

AWS Machine Learning Specialty Exam

View Set

Business Law Ch 13-1 & 13-2 Terms

View Set